Rotations in Bloch Sphere about an arbitrary axis

AI Thread Summary
The discussion focuses on understanding the rotation of a qubit state represented by the Bloch vector in relation to an arbitrary axis on the Bloch sphere. The rotation operator R_n(θ) is defined in terms of the Pauli matrices, and its expression can be derived using Taylor expansion. The challenge lies in proving that R_n(θ) represents a true rotation about the n-axis by the angle θ. A suggested approach involves comparing the Bloch vectors before and after the rotation, ensuring they maintain the same magnitude and that their projections onto the plane perpendicular to n can help determine the angle of rotation. This method aims to solidify the understanding of R_n(θ) as a legitimate rotation operator.
polyChron
Messages
2
Reaction score
0
Hey,
(I have already asked the question at http://physics.stackexchange.com/questions/244586/bloch-sphere-interpretation-of-rotations, I am not sure this forum's etiquette allows that!)

I am trying to understand the following statement. "Suppose a single qubit has a state represented by the Bloch vector ##\vec{\lambda}##. Then the effect of the rotation ##R_{\hat{n}}(\theta)## on the state is to rotate it by an angle $\theta$ about the ##\hat{n}## axis of the Bloch sphere. This fact explains the rather mysterious looking factor of two in the definition of the rotation matrices."
I could work out that the rotation operators ##R_x(\theta)##, ##R_y(\theta)## and ##R_z(\theta)## are infact rotations about the ##X,Y## and ##Z## axis. But how do I extend this for ##R_{\hat{n}}(\theta)## and prove the above statement. Please point me in the right direction.

Thanks.
 
Physics news on Phys.org
I think what you want is what the pauli spin observable is along the "\hat{n}" axis.
Where \hat{n} is a unit vector with components (n_{x},n_{y},n_{z}),

\sigma_{\hat{n}} = \sigma_{x}n_{x} + \sigma_{y} n_{y} + \sigma_{z} n_{z}.

If you know what R_{x}(\theta) is in terms of \sigma_{x}, then I think you can work out what R_{\hat{n}}(\theta) is in terms of \sigma_{\hat{n}}.
 
jfizzix said:
I think what you want is what the pauli spin observable is along the "\hat{n}" axis.
Where \hat{n} is a unit vector with components (n_{x},n_{y},n_{z}),

\sigma_{\hat{n}} = \sigma_{x}n_{x} + \sigma_{y} n_{y} + \sigma_{z} n_{z}.

If you know what R_{x}(\theta) is in terms of \sigma_{x}, then I think you can work out what R_{\hat{n}}(\theta) is in terms of \sigma_{\hat{n}}.

Thanks for your reply.
Yes I have figured that out as well. Let me explain.
I know the rotation matrices in terms of the Pauli matrices, i.e R_x(\theta) = e^{-i\sigma_x /2} and the rotation matrices for \sigma_y and \sigma_z follows in the same manner. I could also prove that R_{\hat{n}}(\theta) = cos(\frac{\theta}{2})I - i sin(\frac{\theta}{2})(n_x\sigma_x+n_y\sigma_y+n_z\sigma_z) using the Taylor expansion. But the difficulties for me start from here. How do I show that R_{\hat{n}}(\theta) is infact a rotation about \hat{n} axis by \theta. How can I construct a concrete proof?
 
polyChron said:
Thanks for your reply.
Yes I have figured that out as well. Let me explain.
I know the rotation matrices in terms of the Pauli matrices, i.e R_x(\theta) = e^{-i\sigma_x /2} and the rotation matrices for \sigma_y and \sigma_z follows in the same manner. I could also prove that R_{\hat{n}}(\theta) = cos(\frac{\theta}{2})I - i sin(\frac{\theta}{2})(n_x\sigma_x+n_y\sigma_y+n_z\sigma_z) using the Taylor expansion. But the difficulties for me start from here. How do I show that R_{\hat{n}}(\theta) is infact a rotation about \hat{n} axis by \theta. How can I construct a concrete proof?

I think what you should do is a proof by demonstration. Compare a bloch vector before \hat{u} and after \hat{u}' a rotation about the n-axis.
Both \hat{u} and \hat{u}' dotted with \hat{n} should give the same value, and the vectors themselves should have the same magnitude. This proves that R_{n}(\theta) is at least some sort of rotation about the \hat{n}-axis.

To find the angle, you need to project \hat{u} and \hat{u}' onto the plane perpendicular to \hat{n}. The dot product of these projected vectors will be the magnitude of each times the cosine of the angle between them, and hopefully that angle will be none other than \theta.
 
The rope is tied into the person (the load of 200 pounds) and the rope goes up from the person to a fixed pulley and back down to his hands. He hauls the rope to suspend himself in the air. What is the mechanical advantage of the system? The person will indeed only have to lift half of his body weight (roughly 100 pounds) because he now lessened the load by that same amount. This APPEARS to be a 2:1 because he can hold himself with half the force, but my question is: is that mechanical...
Some physics textbook writer told me that Newton's first law applies only on bodies that feel no interactions at all. He said that if a body is on rest or moves in constant velocity, there is no external force acting on it. But I have heard another form of the law that says the net force acting on a body must be zero. This means there is interactions involved after all. So which one is correct?
Thread 'Beam on an inclined plane'
Hello! I have a question regarding a beam on an inclined plane. I was considering a beam resting on two supports attached to an inclined plane. I was almost sure that the lower support must be more loaded. My imagination about this problem is shown in the picture below. Here is how I wrote the condition of equilibrium forces: $$ \begin{cases} F_{g\parallel}=F_{t1}+F_{t2}, \\ F_{g\perp}=F_{r1}+F_{r2} \end{cases}. $$ On the other hand...
Back
Top